If $G$ has a nontrivial centre, must every subgroup of index $3$ be normal?












3












$begingroup$



If a group $G$ has a nontrivial centre, must every subgroup of index $3$ be normal?




$S_3$ yields an example of a group with a non-normal subgroup of index $3$, although it has a trivial centre. Moreover, for finite $G$, it's well-known that if $p$ is the smallest prime dividing $|G|$, then any subgroup of index $p$ is normal. Hence the answer to this question is "yes" if $G$ is a finite group of odd order divisible by $3$.



I'm considering dihedral groups as possible counterexamples, but haven't come up with anything.










share|cite|improve this question











$endgroup$








  • 3




    $begingroup$
    Can’t you fancy up your permutation example into a counterexample by direct products?
    $endgroup$
    – Randall
    Dec 31 '18 at 0:25










  • $begingroup$
    Randall's correct; you can just take $S_3 times C_2$.
    $endgroup$
    – Qiaochu Yuan
    Dec 31 '18 at 0:40










  • $begingroup$
    @Randall Thanks. Feel free to post an answer so I can accept it.
    $endgroup$
    – MathematicsStudent1122
    Dec 31 '18 at 0:45










  • $begingroup$
    @MathematicsStudent1122 nah you go for it. It’s good for you.
    $endgroup$
    – Randall
    Dec 31 '18 at 1:10










  • $begingroup$
    The statement is true however, for every non-trivial 3-group ...
    $endgroup$
    – Nicky Hekster
    Dec 31 '18 at 10:34
















3












$begingroup$



If a group $G$ has a nontrivial centre, must every subgroup of index $3$ be normal?




$S_3$ yields an example of a group with a non-normal subgroup of index $3$, although it has a trivial centre. Moreover, for finite $G$, it's well-known that if $p$ is the smallest prime dividing $|G|$, then any subgroup of index $p$ is normal. Hence the answer to this question is "yes" if $G$ is a finite group of odd order divisible by $3$.



I'm considering dihedral groups as possible counterexamples, but haven't come up with anything.










share|cite|improve this question











$endgroup$








  • 3




    $begingroup$
    Can’t you fancy up your permutation example into a counterexample by direct products?
    $endgroup$
    – Randall
    Dec 31 '18 at 0:25










  • $begingroup$
    Randall's correct; you can just take $S_3 times C_2$.
    $endgroup$
    – Qiaochu Yuan
    Dec 31 '18 at 0:40










  • $begingroup$
    @Randall Thanks. Feel free to post an answer so I can accept it.
    $endgroup$
    – MathematicsStudent1122
    Dec 31 '18 at 0:45










  • $begingroup$
    @MathematicsStudent1122 nah you go for it. It’s good for you.
    $endgroup$
    – Randall
    Dec 31 '18 at 1:10










  • $begingroup$
    The statement is true however, for every non-trivial 3-group ...
    $endgroup$
    – Nicky Hekster
    Dec 31 '18 at 10:34














3












3








3





$begingroup$



If a group $G$ has a nontrivial centre, must every subgroup of index $3$ be normal?




$S_3$ yields an example of a group with a non-normal subgroup of index $3$, although it has a trivial centre. Moreover, for finite $G$, it's well-known that if $p$ is the smallest prime dividing $|G|$, then any subgroup of index $p$ is normal. Hence the answer to this question is "yes" if $G$ is a finite group of odd order divisible by $3$.



I'm considering dihedral groups as possible counterexamples, but haven't come up with anything.










share|cite|improve this question











$endgroup$





If a group $G$ has a nontrivial centre, must every subgroup of index $3$ be normal?




$S_3$ yields an example of a group with a non-normal subgroup of index $3$, although it has a trivial centre. Moreover, for finite $G$, it's well-known that if $p$ is the smallest prime dividing $|G|$, then any subgroup of index $p$ is normal. Hence the answer to this question is "yes" if $G$ is a finite group of odd order divisible by $3$.



I'm considering dihedral groups as possible counterexamples, but haven't come up with anything.







group-theory normal-subgroups dihedral-groups






share|cite|improve this question















share|cite|improve this question













share|cite|improve this question




share|cite|improve this question








edited Dec 30 '18 at 23:51









Shaun

9,789113684




9,789113684










asked Dec 30 '18 at 23:47









MathematicsStudent1122MathematicsStudent1122

8,98332668




8,98332668








  • 3




    $begingroup$
    Can’t you fancy up your permutation example into a counterexample by direct products?
    $endgroup$
    – Randall
    Dec 31 '18 at 0:25










  • $begingroup$
    Randall's correct; you can just take $S_3 times C_2$.
    $endgroup$
    – Qiaochu Yuan
    Dec 31 '18 at 0:40










  • $begingroup$
    @Randall Thanks. Feel free to post an answer so I can accept it.
    $endgroup$
    – MathematicsStudent1122
    Dec 31 '18 at 0:45










  • $begingroup$
    @MathematicsStudent1122 nah you go for it. It’s good for you.
    $endgroup$
    – Randall
    Dec 31 '18 at 1:10










  • $begingroup$
    The statement is true however, for every non-trivial 3-group ...
    $endgroup$
    – Nicky Hekster
    Dec 31 '18 at 10:34














  • 3




    $begingroup$
    Can’t you fancy up your permutation example into a counterexample by direct products?
    $endgroup$
    – Randall
    Dec 31 '18 at 0:25










  • $begingroup$
    Randall's correct; you can just take $S_3 times C_2$.
    $endgroup$
    – Qiaochu Yuan
    Dec 31 '18 at 0:40










  • $begingroup$
    @Randall Thanks. Feel free to post an answer so I can accept it.
    $endgroup$
    – MathematicsStudent1122
    Dec 31 '18 at 0:45










  • $begingroup$
    @MathematicsStudent1122 nah you go for it. It’s good for you.
    $endgroup$
    – Randall
    Dec 31 '18 at 1:10










  • $begingroup$
    The statement is true however, for every non-trivial 3-group ...
    $endgroup$
    – Nicky Hekster
    Dec 31 '18 at 10:34








3




3




$begingroup$
Can’t you fancy up your permutation example into a counterexample by direct products?
$endgroup$
– Randall
Dec 31 '18 at 0:25




$begingroup$
Can’t you fancy up your permutation example into a counterexample by direct products?
$endgroup$
– Randall
Dec 31 '18 at 0:25












$begingroup$
Randall's correct; you can just take $S_3 times C_2$.
$endgroup$
– Qiaochu Yuan
Dec 31 '18 at 0:40




$begingroup$
Randall's correct; you can just take $S_3 times C_2$.
$endgroup$
– Qiaochu Yuan
Dec 31 '18 at 0:40












$begingroup$
@Randall Thanks. Feel free to post an answer so I can accept it.
$endgroup$
– MathematicsStudent1122
Dec 31 '18 at 0:45




$begingroup$
@Randall Thanks. Feel free to post an answer so I can accept it.
$endgroup$
– MathematicsStudent1122
Dec 31 '18 at 0:45












$begingroup$
@MathematicsStudent1122 nah you go for it. It’s good for you.
$endgroup$
– Randall
Dec 31 '18 at 1:10




$begingroup$
@MathematicsStudent1122 nah you go for it. It’s good for you.
$endgroup$
– Randall
Dec 31 '18 at 1:10












$begingroup$
The statement is true however, for every non-trivial 3-group ...
$endgroup$
– Nicky Hekster
Dec 31 '18 at 10:34




$begingroup$
The statement is true however, for every non-trivial 3-group ...
$endgroup$
– Nicky Hekster
Dec 31 '18 at 10:34










0






active

oldest

votes











Your Answer





StackExchange.ifUsing("editor", function () {
return StackExchange.using("mathjaxEditing", function () {
StackExchange.MarkdownEditor.creationCallbacks.add(function (editor, postfix) {
StackExchange.mathjaxEditing.prepareWmdForMathJax(editor, postfix, [["$", "$"], ["\\(","\\)"]]);
});
});
}, "mathjax-editing");

StackExchange.ready(function() {
var channelOptions = {
tags: "".split(" "),
id: "69"
};
initTagRenderer("".split(" "), "".split(" "), channelOptions);

StackExchange.using("externalEditor", function() {
// Have to fire editor after snippets, if snippets enabled
if (StackExchange.settings.snippets.snippetsEnabled) {
StackExchange.using("snippets", function() {
createEditor();
});
}
else {
createEditor();
}
});

function createEditor() {
StackExchange.prepareEditor({
heartbeatType: 'answer',
autoActivateHeartbeat: false,
convertImagesToLinks: true,
noModals: true,
showLowRepImageUploadWarning: true,
reputationToPostImages: 10,
bindNavPrevention: true,
postfix: "",
imageUploader: {
brandingHtml: "Powered by u003ca class="icon-imgur-white" href="https://imgur.com/"u003eu003c/au003e",
contentPolicyHtml: "User contributions licensed under u003ca href="https://creativecommons.org/licenses/by-sa/3.0/"u003ecc by-sa 3.0 with attribution requiredu003c/au003e u003ca href="https://stackoverflow.com/legal/content-policy"u003e(content policy)u003c/au003e",
allowUrls: true
},
noCode: true, onDemand: true,
discardSelector: ".discard-answer"
,immediatelyShowMarkdownHelp:true
});


}
});














draft saved

draft discarded


















StackExchange.ready(
function () {
StackExchange.openid.initPostLogin('.new-post-login', 'https%3a%2f%2fmath.stackexchange.com%2fquestions%2f3057290%2fif-g-has-a-nontrivial-centre-must-every-subgroup-of-index-3-be-normal%23new-answer', 'question_page');
}
);

Post as a guest















Required, but never shown

























0






active

oldest

votes








0






active

oldest

votes









active

oldest

votes






active

oldest

votes
















draft saved

draft discarded




















































Thanks for contributing an answer to Mathematics Stack Exchange!


  • Please be sure to answer the question. Provide details and share your research!

But avoid



  • Asking for help, clarification, or responding to other answers.

  • Making statements based on opinion; back them up with references or personal experience.


Use MathJax to format equations. MathJax reference.


To learn more, see our tips on writing great answers.




draft saved


draft discarded














StackExchange.ready(
function () {
StackExchange.openid.initPostLogin('.new-post-login', 'https%3a%2f%2fmath.stackexchange.com%2fquestions%2f3057290%2fif-g-has-a-nontrivial-centre-must-every-subgroup-of-index-3-be-normal%23new-answer', 'question_page');
}
);

Post as a guest















Required, but never shown





















































Required, but never shown














Required, but never shown












Required, but never shown







Required, but never shown

































Required, but never shown














Required, but never shown












Required, but never shown







Required, but never shown







Popular posts from this blog

How do I know what Microsoft account the skydrive app is syncing to?

When does type information flow backwards in C++?

Grease: Live!